Übung Nr. 12
Abgabetermin: Mittwoch, den 30. Januar 2001
Aufgabe 1: (6 Punkte)
Die Gangdifferenz zwischen dem ersten an der Oberseite und dem ersten an der Unterseite reflektierten Strahl beträgt

\begin{displaymath}
\Delta = n ( \overline{BC} + \overline{CA}) - \left( \overline{FA}
+ \frac{\lambda}{2} \right).
\end{displaymath}


Hierfür können wir auch schreiben

\begin{displaymath}
\Delta = 2n \frac{d}{cos\beta} - 2 d \; tg\beta \; sin\alpha
+ \frac{\lambda}{2}.
\end{displaymath}

Mit $sin\beta = (sin\alpha)/n$ und $cos\beta = \sqrt{1 -
(sin^{2}\alpha)/n^{2}}$ folgt:

\begin{displaymath}
\Delta = \frac{2nd}{\sqrt{1 - (sin^{2}\alpha)/n^{2}}} -
\f...
...^{2}}{\sqrt{1 - (sin^{2}\alpha)/n^{2}}}
+ \frac{\lambda}{2}.
\end{displaymath}

Dieses kann man umformen zu

\begin{displaymath}
\Delta = 2 d \sqrt{n^{2}-sin^{2}\alpha} + \frac{\lambda}{2}.
\end{displaymath}

Man erhält Auslöschung, wenn diese Gangdifferenz gleich $(k+1/2)\lambda$ mit ganzer Zahl $k$ ist. Also folgt

\begin{displaymath}
d = k \frac{\lambda}{2\sqrt{n^{2}-sin^{2}\alpha}} = k \cdot...
...4 \cdot
10^{-5} \; cm, \; \; \; \; \; \; \; \; k=1,2,3,4,...
\end{displaymath}


Aufgabe 2: (7 Punkte)
a) Die Intensitätsverteilung am unendlich langen Einzelspalt ist nach Vorlesung

\begin{displaymath}
I = I_{0} \frac{sin^{2}\varphi}{\varphi^{2}},
\end{displaymath}

mit $\varphi = (kb/2)sin\alpha = (\pi b/\lambda) sin\alpha$. Extremwerte ergeben sich für

\begin{displaymath}
\frac{dI}{d\varphi} = I_{0} \frac{2\varphi^{2} \; sin^{2}\v...
... \; (\varphi \; cos\varphi - sin\varphi)}
{\varphi^{3}} = 0.
\end{displaymath}

Minima ergeben sich für $sin\varphi = 0$ und $\varphi \neq 0$, d.h.

\begin{displaymath}
\varphi_{min,k} = \pm k \pi, \; \; \; \; \; k=1,2,3,....
\end{displaymath}

Maxima erhält man entsprechend für $\varphi \; cos\varphi - sin\varphi = 0$ oder $\varphi = tg\varphi$. Dieses wird erfüllt durch
$\displaystyle \varphi_{max,1}$ $\textstyle =$ $\displaystyle \pm 1,43 \pi$  
$\displaystyle \varphi_{max,2}$ $\textstyle =$ $\displaystyle \pm 2,46 \pi$  
$\displaystyle \varphi_{max,3}$ $\textstyle =$ $\displaystyle \pm 3,47 \pi$  

Die Maxima liegen also nicht exakt zwischen den Minima, sondern geringfügig zu kleineren Winkeln verschoben. Wir können das noch durch die wahren Ablenkwinkel ausdrücken. Wegen $\alpha \approx sin\alpha
= 2\varphi/(kb) = \varphi \lambda/(\pi b)$ folgt für die Minima

\begin{displaymath}
\alpha_{min,k} = \pm k \pi \frac{\lambda}{\pi b} = \pm k \cdot 5 \cdot
10^{-4}, \; \; \; \; \; k=1,2,3,....
\end{displaymath}

und für die ersten drei Nebenmaxima:
$\displaystyle \alpha_{max,1}$ $\textstyle \approx$ $\displaystyle \pm 7,14 \cdot 10^{-4}$  
$\displaystyle \alpha_{max,2}$ $\textstyle \approx$ $\displaystyle \pm 1,22 \cdot 10^{-3}$  
$\displaystyle \alpha_{max,3}$ $\textstyle \approx$ $\displaystyle \pm 1,73 \cdot 10^{-3}$  

b) Wie wir in Teil a) gesehen haben, liegen die Maxima zwar nicht exakt zwischen den Minima, sind aber nur geringfügig verschoben. Zur Berechnung der Intensität in den Maxima können wir daher setzen

\begin{displaymath}
\varphi_{max,k} \approx \pm \left( k + \frac{1}{2} \right) \pi, \; \; \; \; \;
k = 1,2,3,....
\end{displaymath}

Dann ist aber $sin\varphi_{max,k} = 1$ und daher

\begin{displaymath}
I_{max,k} = \frac{I_{0}}{\varphi^{2}_{max,k}} = \frac{I_{0}}
{[(k+1/2)\pi]^{2}}
\end{displaymath}

c) Die Intensität des Hauptmaximums fällt auf die Hälfte ihres Wertes ab, wenn

\begin{displaymath}
\frac{I(\alpha)}{I_{0}} = \frac{sin^{2}\varphi}{\varphi^{2}} = \frac{1}{2}
\end{displaymath}

oder $sin\varphi = 0,707 \; \varphi$. Diese Gleichung wird gelöst durch $\varphi_{1/2} \approx 1,39$. Da $\varphi = (\pi b/\lambda) sin\alpha
\approx (\pi b/\lambda) \alpha$, folgt

\begin{displaymath}
\alpha_{1/2} = \frac{\lambda}{\pi b} \varphi_{1/2}
\end{displaymath}

und

\begin{displaymath}
\Delta \alpha_{1/2} = 2 \alpha_{1/2} = 0,885 \frac{\lambda}{b}
\approx \frac{\lambda}{b}.
\end{displaymath}


Aufgabe 3: (7 Punkte)
Die Feldstärke der drei Wellen kann geschrieben werden als (siehe Skript)

\begin{displaymath}
E(\delta) = C + C e^{i\delta} + C e^{i(5/2)\delta},
\; \; \; \; \; \; \; \; \; \delta = \frac{2\pi d}{\lambda} sin\alpha.
\end{displaymath}

Die Intensität ist

\begin{displaymath}
I(\delta) = E E^{\ast} = C^{2}\left( 1+e^{i\delta} + e^{i(5/...
...} \right)
\left( 1 + e^{-i\delta} + e^{-i(5/2)\delta} \right).
\end{displaymath}

Ausmultiplizieren und mit Hilfe der Formel $cos(x) = (1/2)(e^{ix} + e^{-ix})$ folgt

\begin{displaymath}
I(\delta) = C^{2} \left[ 3 + 2 \; cos(\delta) + 2 \; cos\lef...
...a
\right) + 2 \; cos \left( \frac{5}{2}\delta \right) \right].
\end{displaymath}

Die Hauptmaximam liegen bei den Winkeln, bei denen die Phasen aller drei Teilwellen übereinstimmen. Bezeichnen wir den Wegunterschied der zweiten und dritten Welle gegeüber der ersten mit $h_{1}$ und $h_{2}$, so ist

\begin{displaymath}
sin\alpha = \frac{h_{1}}{d} = \frac{h_{2}}{d+(3/2)d} = \frac{h_{2}}{(5/2)d}
\end{displaymath}

Da $n=1$, ist der Wegunterschied identisch mit dem optischen Gangunterschied. Maximale Verstärkung erhalten wir, wenn gleichzeitig

$\displaystyle h_{1}$ $\textstyle =$ $\displaystyle m_{1} \lambda = d \; sin\alpha$  
$\displaystyle h_{2}$ $\textstyle =$ $\displaystyle m_{2} \lambda = (5/2) \; sin\alpha,$  

wobei $m_{1}$ und $m_{2}$ ganze Zahlen sind. Dieses kann nur dann gleichzeitig erfüllt werden, wenn

\begin{displaymath}
sin\alpha = \frac{m_{1}\lambda}{d} = \frac{m_{2}\lambda}{(5/2)d}
\end{displaymath}

gilt, d.h. $5 m_{1} = 2 m_{2}$. Die einfachste, von Null verschiedene Lösung ist $m_{1}=2$ und $m_{2} = 5$. Das erste Hauptmaximum liegt also unter dem Winkel $sin\alpha = 2\lambda /d$ oder $\delta = 4\pi$. Allgemein gilt:

\begin{displaymath}
sin\alpha_{m} = \pm m \; \frac{2\lambda}{d}, \; \; \; \; m=0,1,2,3,....
\end{displaymath}

Für die Intensitäten in den Maxima gilt

\begin{displaymath}
I(\delta = 0) = I(\delta = 4\pi) = 9 C^{2} = I_{0}.
\end{displaymath}

Einem Winkel von $\alpha = \alpha_{1}/2$ entsprecht $\delta = 2\pi$. Hierfür erhalten wir:

\begin{displaymath}
I(\delta = 2\pi) = C^{2}(3 + 2 - 2 - 2) = C^{2}
\end{displaymath}

und

\begin{displaymath}
I(\delta = 2\pi) = \frac{I_{0}}{9}.
\end{displaymath}





Harm Fesefeldt
2008-01-11